Difference between revisions of "2020 AMC 10B Problems/Problem 21"

(Solution)
(Redirected page to 2020 AMC 12B Problems/Problem 18)
(Tag: New redirect)
(32 intermediate revisions by 17 users not shown)
Line 1: Line 1:
==Problem==
+
#redirect [[2020 AMC 12B Problems/Problem 18]]
 
 
In square <math>ABCD</math>, points <math>E</math> and <math>H</math> lie on <math>\overline{AB}</math> and <math>\overline{DA}</math>, respectively, so that <math>AE=AH.</math> Points <math>F</math> and <math>G</math> lie on <math>\overline{BC}</math> and <math>\overline{CD}</math>, respectively, and points <math>I</math> and <math>J</math> lie on <math>\overline{EH}</math> so that <math>\overline{FI} \perp \overline{EH}</math> and <math>\overline{GJ} \perp \overline{EH}</math>. See the figure below. Triangle <math>AEH</math>, quadrilateral <math>BFIE</math>, quadrilateral <math>DHJG</math>, and pentagon <math>FCGJI</math> each has area <math>1.</math> What is <math>FI^2</math>?
 
<asy>
 
real x=2sqrt(2);
 
real y=2sqrt(16-8sqrt(2))-4+2sqrt(2);
 
real z=2sqrt(8-4sqrt(2));
 
pair A, B, C, D, E, F, G, H, I, J;
 
A = (0,0);
 
B = (4,0);
 
C = (4,4);
 
D = (0,4);
 
E = (x,0);
 
F = (4,y);
 
G = (y,4);
 
H = (0,x);
 
I = F + z * dir(225);
 
J = G + z * dir(225);
 
 
 
draw(A--B--C--D--A);
 
draw(H--E);
 
draw(J--G^^F--I);
 
draw(rightanglemark(G, J, I), linewidth(.5));
 
draw(rightanglemark(F, I, E), linewidth(.5));
 
 
 
dot("$A$", A, S);
 
dot("$B$", B, S);
 
dot("$C$", C, dir(90));
 
dot("$D$", D, dir(90));
 
dot("$E$", E, S);
 
dot("$F$", F, dir(0));
 
dot("$G$", G, N);
 
dot("$H$", H, W);
 
dot("$I$", I, SW);
 
dot("$J$", J, SW);
 
 
 
</asy>
 
<math>\textbf{(A) } \frac{7}{3} \qquad \textbf{(B) } 8-4\sqrt2 \qquad \textbf{(C) } 1+\sqrt2 \qquad \textbf{(D) } \frac{7}{4}\sqrt2 \qquad \textbf{(E) } 2\sqrt2</math>
 
 
 
==Solution==
 
Since the total area is <math>4</math>, the side length of square <math>ABCD</math> is <math>2</math>. We see that since triangle <math>HAE</math> is a right isosceles triangle with area 1, we can determine sides <math>HA</math> and <math>AE</math> both to be <math>\sqrt{2}</math>. Now, consider extending <math>FB</math> and <math>IE</math> until they intersect. Let the point of intersection be <math>K</math>. We note that <math>EBK</math> is also a right isosceles triangle with side <math>2-\sqrt{2}</math> and find it's area to be <math>3-\sqrt{2}</math>. Now, we notice that <math>FIK</math> is also a right isosceles triangle and find it's area to be <math>\frac{1}{2}</math><math>FI^2</math>. This is also equal to <math>1+3-2\sqrt{2}</math> or <math>4-2\sqrt{2}</math>. Since we are looking for <math>FI^2</math>, we want two times this. That gives <math>\boxed{\textbf{(B)}\ 8-4\sqrt{2}}</math>
 
 
 
==See Also==
 
 
 
{{AMC10 box|year=2020|ab=B|num-b=20|num-a=22}}
 
{{MAA Notice}}
 

Revision as of 17:57, 9 December 2020